TransWikia.com

How to understand this formula in quantum field theory?

Physics Asked on March 3, 2021

In Condensed Matter Field Theory (page 288) by Altland and Simons, there is one mathematical formula:

begin{equation}
sum_mathbf{p} left(mathbf{p}boldsymbol{cdot}mathbf{v}right)left(mathbf{p}boldsymbol{cdot}mathbf{v}’right)Fleft(mathbf{p}^2right)boldsymbol{=}dfrac{mathbf{v}boldsymbol{cdot}mathbf{v}’}{d}sum_mathbf{p}:mathbf{p}^2Fleft(mathbf{p}^2right)
tag{01}label{01}
end{equation}

Here, $mathbf{p}$ is the momentum, and $mathbf{v}$ and $mathbf{v}’$ are vectors. $Fleft(mathbf{p}^2right)$ is a function of momentum which is invariant under rotational transformations; $d$ is the dimension considered (e.g., $d=3$ in $x,y,z$ coordinate system).
My question is how to arrive at this formula?

Mathematically, this formula seems not correct. For example, imagine a summation over only $mathbf{p}boldsymbol{=}mathbf{p}_0$ and imagine $mathbf{v}$ and $mathbf{v}’$ being orthogonal: then the left hand side is $left(mathbf{p}_0boldsymbol{cdot}mathbf{v}right)left(mathbf{p}_0boldsymbol{cdot}mathbf{v}’right)Fleft(mathbf{p}_0^2right)$ which can be non zero, while the right hand side is zero because $left(mathbf{v}boldsymbol{cdot}mathbf{v}’right)boldsymbol{=}0$. I am wondering if there are some arguments in physics to "prove" this formula. For example, if the summation is replaced by an integration over the whole reciprocal space of momentum p with d dimension, will the formula stand?

One Answer

The formula holds, because there is a sum over all ${bf p}$. After that sum is performed, ${bf v}cdot{bf v}'$ is essentially the only vector structure that is consistent with rotation symmetry.

To prove the formula, first rewrite it so that the ${bf v}$ and ${bf v}$' are outside the sum: $$sum_{{bf p}}({bf p}cdot{bf v})({bf p}cdot{bf v}')F(p^{2})= sum_{j=1}^{d}v_{j}sum_{j=1}^{d}v_{k}'sum_{{bf p}}p_{j}p_{k}F(p^{2}).$$

Now look just at the final sum, $sum_{{bf p}}p_{j}p_{k}F(p^{2})$. Consider holding the index $j$ fixed (say $j=1$) and varying $k$. If $kneq j$ (say $j=2$) then for every term in the sum with ${bf p}=(p_{j},p_{k},p_{3})$, there is another term in the sum with ${bf p}'=(p_{j},-p_{k},p_{3})$. Since the sum terms $p_{j}p_{k}F(p^{2})$ and $p_{j}'p_{k}'F(p'^{2})$ are equal in magnitude but opposite in sign, they will cancel; and, if fact, every term in the sum will be canceled by a similar one with an inverted $p_{k}rightarrow p_{k}'=-p_{k}$. Thus, the sum is $sum_{{bf p}}p_{j}p_{k}F(p^{2})=0$ if $jneq k$.

The expression $sum_{{bf p}}p_{j}p_{k}F(p^{2})$ transforms like a two-index tensor. (Any product of the components of two vectors ${bf u}$ and ${bf w}$ along with a scalar $phi$—that is, $u_{j}w_{k}phi$—transforms this way. The sum of interest is then just a sum of tensors that transform this way and so transforms this way itself.) The only two-tensor that has components that are nonzero only for $j=k$ is one that is proportional to the identity, with components $delta_{jk}$. [This is essentially just a long-winded way of saying that, since there is no preferred direction, $sum_{{bf p}}p_{1}p_{1}F(p^{2}) =sum_{{bf p}}p_{2}p_{2}F(p^{2})=sum_{{bf p}}p_{3}p_{3}F(p^{2})$.] So we must have $sum_{{bf p}}p_{j}p_{k}F(p^{2})$ be a scalar function times $delta_{jk}$, $$sum_{{bf p}}p_{j}p_{k}F(p^{2})=delta_{jk}G,$$ which leaves us to find $G$.

To get the scalar $G$, we contract the tensor with another $delta_{jk}$, or $$sum_{j=1}^{d}sum_{k=1}^{d}delta_{jk}sum_{{bf p}}p_{j}p_{k}F(p^{2})=sum_{j=1}^{d}sum_{k=1}^{d}delta_{jk}delta_{jk}G,$$ which reduces to $$sum_{{bf p}}p^{2}F(p^{2})=dG,$$ since $sum_{j=1}^{d}sum_{k=1}^{d}delta_{jk}delta_{jk}$ is just the dimensionality of space $d$.

Then it is just a matter of inserting this expression into the earlier ones and working backwards. Solving for $G$, we have $$sum_{{bf p}}p_{j}p_{k}F(p^{2})=delta_{jk}G=delta_{jk}frac{1}{d}sum_{{bf p}}p^{2}F(p^{2}).$$ Then inserting this into the original expression with $v_{j}$ and $v_{k}'$ gives $$sum_{j=1}^{d}v_{j}sum_{j=1}^{d}v_{k}'sum_{{bf p}}p_{j}p_{k}F(p^{2})= sum_{j=1}^{d}v_{j}sum_{j=1}^{d}v_{k}'delta_{jk}frac{1}{d}sum_{{bf p}}p^{2}F(p^{2}),$$ and collecting this in component-free form gives the final expression $$sum_{{bf p}}({bf p}cdot{bf v})({bf p}cdot{bf v}')F(p^{2})=frac{1}{d}{bf v}cdot{bf v}'sum_{{bf p}}p^{2}F(p^{2}).$$

Correct answer by Buzz on March 3, 2021

Add your own answers!

Ask a Question

Get help from others!

© 2024 TransWikia.com. All rights reserved. Sites we Love: PCI Database, UKBizDB, Menu Kuliner, Sharing RPP